Upper bound for the number of solutions of a Diophantine equationInfinitely many solutions of a diophantine equationLogarithmic bound for Diophantine equationQuantum Mechanics derivation of Wallis' Formula?Solvability of Diophantine equation using congruences where the variables are bounded?Is there (conjectural) upper bound for the largest solution of Diophantine equation with finitely many solutions?All the solutions of linear Diophantine equationOn the notion of primary representation of a natural number by a quadratic formCase D=4l in Elkies' paper on Supersingular Primes of an Elliptic Curve over $mathbbQ$Best possibe bound for the number of solutions of diophantine approximation?Solutions to diophantine equation

Upper bound for the number of solutions of a Diophantine equation


Infinitely many solutions of a diophantine equationLogarithmic bound for Diophantine equationQuantum Mechanics derivation of Wallis' Formula?Solvability of Diophantine equation using congruences where the variables are bounded?Is there (conjectural) upper bound for the largest solution of Diophantine equation with finitely many solutions?All the solutions of linear Diophantine equationOn the notion of primary representation of a natural number by a quadratic formCase D=4l in Elkies' paper on Supersingular Primes of an Elliptic Curve over $mathbbQ$Best possibe bound for the number of solutions of diophantine approximation?Solutions to diophantine equation













1












$begingroup$


Consider the Diophantine equation



$$k^2 + k - sigma (ell^2 + ell) = m,$$
where $N leq k leq 2 N$, $L leq ell leq 2 L$, $m in mathbbZ$ and $sigma in mathbbR$.



For which values of the parameter $sigma$ is it possible to say that
the number of solutions of this equation is bounded by $O(min(N, L)^epsilon)$ ?



To facilitate progress, I'm going to put this auxiliar lemma.



$underline Lemma$. For every $varepsilon > 0$, there exists $C_varepsilon > 0$
such that, for every $m in mathbbZ$ and $K$ positive integer,



$$sharp (x, y) in mathbbN^2 mid K leq x leq 2 K , x^2 pm y^2 = m leq C_varepsilon K^varepsilon. $$



I would be very grateful for any suggestion!










share|cite|improve this question









New contributor




Marcelo Nogueira is a new contributor to this site. Take care in asking for clarification, commenting, and answering.
Check out our Code of Conduct.







$endgroup$







  • 10




    $begingroup$
    The lemma is false. Suppose that $m=0$. Then every pair $(x,x)$ with $Kleq xleq 2K$ belongs to the set on the LHS. Hence, this set contains $K+1$ elements and so its cardinality cannot be bounded from above by $C_epsilon K^epsilon$ for $epsilon<1$.
    $endgroup$
    – Philipp Lampe
    8 hours ago







  • 1




    $begingroup$
    The lemma is in fact true. Note that you can take a large constant C > 0 in this case.
    $endgroup$
    – Marcelo Nogueira
    8 hours ago






  • 3




    $begingroup$
    @MarceloNogueira: for what value of $C_1/2$ is $C_1/2K^1/2>K+1$ for all $K$?
    $endgroup$
    – Alex B.
    5 hours ago















1












$begingroup$


Consider the Diophantine equation



$$k^2 + k - sigma (ell^2 + ell) = m,$$
where $N leq k leq 2 N$, $L leq ell leq 2 L$, $m in mathbbZ$ and $sigma in mathbbR$.



For which values of the parameter $sigma$ is it possible to say that
the number of solutions of this equation is bounded by $O(min(N, L)^epsilon)$ ?



To facilitate progress, I'm going to put this auxiliar lemma.



$underline Lemma$. For every $varepsilon > 0$, there exists $C_varepsilon > 0$
such that, for every $m in mathbbZ$ and $K$ positive integer,



$$sharp (x, y) in mathbbN^2 mid K leq x leq 2 K , x^2 pm y^2 = m leq C_varepsilon K^varepsilon. $$



I would be very grateful for any suggestion!










share|cite|improve this question









New contributor




Marcelo Nogueira is a new contributor to this site. Take care in asking for clarification, commenting, and answering.
Check out our Code of Conduct.







$endgroup$







  • 10




    $begingroup$
    The lemma is false. Suppose that $m=0$. Then every pair $(x,x)$ with $Kleq xleq 2K$ belongs to the set on the LHS. Hence, this set contains $K+1$ elements and so its cardinality cannot be bounded from above by $C_epsilon K^epsilon$ for $epsilon<1$.
    $endgroup$
    – Philipp Lampe
    8 hours ago







  • 1




    $begingroup$
    The lemma is in fact true. Note that you can take a large constant C > 0 in this case.
    $endgroup$
    – Marcelo Nogueira
    8 hours ago






  • 3




    $begingroup$
    @MarceloNogueira: for what value of $C_1/2$ is $C_1/2K^1/2>K+1$ for all $K$?
    $endgroup$
    – Alex B.
    5 hours ago













1












1








1


1



$begingroup$


Consider the Diophantine equation



$$k^2 + k - sigma (ell^2 + ell) = m,$$
where $N leq k leq 2 N$, $L leq ell leq 2 L$, $m in mathbbZ$ and $sigma in mathbbR$.



For which values of the parameter $sigma$ is it possible to say that
the number of solutions of this equation is bounded by $O(min(N, L)^epsilon)$ ?



To facilitate progress, I'm going to put this auxiliar lemma.



$underline Lemma$. For every $varepsilon > 0$, there exists $C_varepsilon > 0$
such that, for every $m in mathbbZ$ and $K$ positive integer,



$$sharp (x, y) in mathbbN^2 mid K leq x leq 2 K , x^2 pm y^2 = m leq C_varepsilon K^varepsilon. $$



I would be very grateful for any suggestion!










share|cite|improve this question









New contributor




Marcelo Nogueira is a new contributor to this site. Take care in asking for clarification, commenting, and answering.
Check out our Code of Conduct.







$endgroup$




Consider the Diophantine equation



$$k^2 + k - sigma (ell^2 + ell) = m,$$
where $N leq k leq 2 N$, $L leq ell leq 2 L$, $m in mathbbZ$ and $sigma in mathbbR$.



For which values of the parameter $sigma$ is it possible to say that
the number of solutions of this equation is bounded by $O(min(N, L)^epsilon)$ ?



To facilitate progress, I'm going to put this auxiliar lemma.



$underline Lemma$. For every $varepsilon > 0$, there exists $C_varepsilon > 0$
such that, for every $m in mathbbZ$ and $K$ positive integer,



$$sharp (x, y) in mathbbN^2 mid K leq x leq 2 K , x^2 pm y^2 = m leq C_varepsilon K^varepsilon. $$



I would be very grateful for any suggestion!







nt.number-theory diophantine-equations






share|cite|improve this question









New contributor




Marcelo Nogueira is a new contributor to this site. Take care in asking for clarification, commenting, and answering.
Check out our Code of Conduct.











share|cite|improve this question









New contributor




Marcelo Nogueira is a new contributor to this site. Take care in asking for clarification, commenting, and answering.
Check out our Code of Conduct.









share|cite|improve this question




share|cite|improve this question








edited 1 hour ago









András Bátkai

3,80642342




3,80642342






New contributor




Marcelo Nogueira is a new contributor to this site. Take care in asking for clarification, commenting, and answering.
Check out our Code of Conduct.









asked 8 hours ago









Marcelo NogueiraMarcelo Nogueira

132




132




New contributor




Marcelo Nogueira is a new contributor to this site. Take care in asking for clarification, commenting, and answering.
Check out our Code of Conduct.





New contributor





Marcelo Nogueira is a new contributor to this site. Take care in asking for clarification, commenting, and answering.
Check out our Code of Conduct.






Marcelo Nogueira is a new contributor to this site. Take care in asking for clarification, commenting, and answering.
Check out our Code of Conduct.







  • 10




    $begingroup$
    The lemma is false. Suppose that $m=0$. Then every pair $(x,x)$ with $Kleq xleq 2K$ belongs to the set on the LHS. Hence, this set contains $K+1$ elements and so its cardinality cannot be bounded from above by $C_epsilon K^epsilon$ for $epsilon<1$.
    $endgroup$
    – Philipp Lampe
    8 hours ago







  • 1




    $begingroup$
    The lemma is in fact true. Note that you can take a large constant C > 0 in this case.
    $endgroup$
    – Marcelo Nogueira
    8 hours ago






  • 3




    $begingroup$
    @MarceloNogueira: for what value of $C_1/2$ is $C_1/2K^1/2>K+1$ for all $K$?
    $endgroup$
    – Alex B.
    5 hours ago












  • 10




    $begingroup$
    The lemma is false. Suppose that $m=0$. Then every pair $(x,x)$ with $Kleq xleq 2K$ belongs to the set on the LHS. Hence, this set contains $K+1$ elements and so its cardinality cannot be bounded from above by $C_epsilon K^epsilon$ for $epsilon<1$.
    $endgroup$
    – Philipp Lampe
    8 hours ago







  • 1




    $begingroup$
    The lemma is in fact true. Note that you can take a large constant C > 0 in this case.
    $endgroup$
    – Marcelo Nogueira
    8 hours ago






  • 3




    $begingroup$
    @MarceloNogueira: for what value of $C_1/2$ is $C_1/2K^1/2>K+1$ for all $K$?
    $endgroup$
    – Alex B.
    5 hours ago







10




10




$begingroup$
The lemma is false. Suppose that $m=0$. Then every pair $(x,x)$ with $Kleq xleq 2K$ belongs to the set on the LHS. Hence, this set contains $K+1$ elements and so its cardinality cannot be bounded from above by $C_epsilon K^epsilon$ for $epsilon<1$.
$endgroup$
– Philipp Lampe
8 hours ago





$begingroup$
The lemma is false. Suppose that $m=0$. Then every pair $(x,x)$ with $Kleq xleq 2K$ belongs to the set on the LHS. Hence, this set contains $K+1$ elements and so its cardinality cannot be bounded from above by $C_epsilon K^epsilon$ for $epsilon<1$.
$endgroup$
– Philipp Lampe
8 hours ago





1




1




$begingroup$
The lemma is in fact true. Note that you can take a large constant C > 0 in this case.
$endgroup$
– Marcelo Nogueira
8 hours ago




$begingroup$
The lemma is in fact true. Note that you can take a large constant C > 0 in this case.
$endgroup$
– Marcelo Nogueira
8 hours ago




3




3




$begingroup$
@MarceloNogueira: for what value of $C_1/2$ is $C_1/2K^1/2>K+1$ for all $K$?
$endgroup$
– Alex B.
5 hours ago




$begingroup$
@MarceloNogueira: for what value of $C_1/2$ is $C_1/2K^1/2>K+1$ for all $K$?
$endgroup$
– Alex B.
5 hours ago










1 Answer
1






active

oldest

votes


















6












$begingroup$

First of all, you can assume $sigmain mathbbQ$. Otherwise, the only solutions there can be are those with $l^2+l=0$ and $k^2+k=m$, and there's certainly only a bounded number of those.



Multiplying by the denominator, we see that we are being asked for a bound on the number of solutions to
$$a (k^2 + k) + b (l^2 + l) = c$$
with $k$, $l$ in dyadic intervals. We multiply both sides by $4$ and add $a+b$ to complete the squares. Thus we reduce our problem to that of bounding the number of solutions to
$$a k^2 + b l^2 = c$$
with $k$, $l$ in dyadic intervals. Multiplying by $a$ and then replacing $a k$ by $k$, we reduce our problem to that of bounding the number of solutions to
$$k^2 + d l^2 = n$$
for given $d$ and $n$,
with $k$ and $l$ in dyadic intervals $K<kleq 2 K$, $L<lleq 2 L$.
(I take the implied constant in the bound you wish is allowed to depend on $sigma$. The bound I will give will depend on $d$, though not on $n$.)



For $d>0$, the number of solutions is obviously bounded by the number of ideals of norm $n$ in the ring of integers of $mathbbQ(sqrtd)$. That number is bounded by the number of divisors of $n$, which is $O_epsilon(n^epsilon) = O_epsilon(max(K,L)^epsilon)$. To obtain the bound $O_epsilon(min(K,L)^epsilon)$, note that, if $L^2 < 2K/3d$, there can be at most one solutions to your equation, as two consecutive values of $k^2$ differ by at least $2 K + 1$, and $d (2 L)^2 - d L^2 = 3 d L^2$.



For $d<0$, you also have to take quadratic units in $mathbbQ(sqrtd)$, but, as there is only a logarithmic number of them in a box of given size (the group of units being isomorphic to $mathbbZ$ times bounded torsion), you still get a bound of $O_epsilon(max(K,L)^epsilon)$, and hence of $O_epsilon(min(K,L)^epsilon)$.



The only exception is given by $n=0$ and $d$ of the form $-r^2$, $r$ an integer. Then the number of solutions to $k^2+d l^2 = n$ is evidently infinite, and the number of solutions in a box is linear on the size of the box. It is easy to see that this is the case of $c=-(a+ b)/4$, $a b=-r^2$ in the equation $a (k^2+k) + b (l^2+ l) = c$. That case corresponds to $sigmain mathbbQ^2$, $m = (sigma-1)/4$ (and thus $sigmain mathbbZ^2$) in the original problem.



tl;dr a simple exercise in quadratic number fields






share|cite|improve this answer










New contributor




Nell is a new contributor to this site. Take care in asking for clarification, commenting, and answering.
Check out our Code of Conduct.






$endgroup$








  • 1




    $begingroup$
    Nell, thanks so much for your explanation !
    $endgroup$
    – Marcelo Nogueira
    4 hours ago











Your Answer





StackExchange.ifUsing("editor", function ()
return StackExchange.using("mathjaxEditing", function ()
StackExchange.MarkdownEditor.creationCallbacks.add(function (editor, postfix)
StackExchange.mathjaxEditing.prepareWmdForMathJax(editor, postfix, [["$", "$"], ["\\(","\\)"]]);
);
);
, "mathjax-editing");

StackExchange.ready(function()
var channelOptions =
tags: "".split(" "),
id: "504"
;
initTagRenderer("".split(" "), "".split(" "), channelOptions);

StackExchange.using("externalEditor", function()
// Have to fire editor after snippets, if snippets enabled
if (StackExchange.settings.snippets.snippetsEnabled)
StackExchange.using("snippets", function()
createEditor();
);

else
createEditor();

);

function createEditor()
StackExchange.prepareEditor(
heartbeatType: 'answer',
autoActivateHeartbeat: false,
convertImagesToLinks: true,
noModals: true,
showLowRepImageUploadWarning: true,
reputationToPostImages: 10,
bindNavPrevention: true,
postfix: "",
imageUploader:
brandingHtml: "Powered by u003ca class="icon-imgur-white" href="https://imgur.com/"u003eu003c/au003e",
contentPolicyHtml: "User contributions licensed under u003ca href="https://creativecommons.org/licenses/by-sa/3.0/"u003ecc by-sa 3.0 with attribution requiredu003c/au003e u003ca href="https://stackoverflow.com/legal/content-policy"u003e(content policy)u003c/au003e",
allowUrls: true
,
noCode: true, onDemand: true,
discardSelector: ".discard-answer"
,immediatelyShowMarkdownHelp:true
);



);






Marcelo Nogueira is a new contributor. Be nice, and check out our Code of Conduct.









draft saved

draft discarded


















StackExchange.ready(
function ()
StackExchange.openid.initPostLogin('.new-post-login', 'https%3a%2f%2fmathoverflow.net%2fquestions%2f325448%2fupper-bound-for-the-number-of-solutions-of-a-diophantine-equation%23new-answer', 'question_page');

);

Post as a guest















Required, but never shown

























1 Answer
1






active

oldest

votes








1 Answer
1






active

oldest

votes









active

oldest

votes






active

oldest

votes









6












$begingroup$

First of all, you can assume $sigmain mathbbQ$. Otherwise, the only solutions there can be are those with $l^2+l=0$ and $k^2+k=m$, and there's certainly only a bounded number of those.



Multiplying by the denominator, we see that we are being asked for a bound on the number of solutions to
$$a (k^2 + k) + b (l^2 + l) = c$$
with $k$, $l$ in dyadic intervals. We multiply both sides by $4$ and add $a+b$ to complete the squares. Thus we reduce our problem to that of bounding the number of solutions to
$$a k^2 + b l^2 = c$$
with $k$, $l$ in dyadic intervals. Multiplying by $a$ and then replacing $a k$ by $k$, we reduce our problem to that of bounding the number of solutions to
$$k^2 + d l^2 = n$$
for given $d$ and $n$,
with $k$ and $l$ in dyadic intervals $K<kleq 2 K$, $L<lleq 2 L$.
(I take the implied constant in the bound you wish is allowed to depend on $sigma$. The bound I will give will depend on $d$, though not on $n$.)



For $d>0$, the number of solutions is obviously bounded by the number of ideals of norm $n$ in the ring of integers of $mathbbQ(sqrtd)$. That number is bounded by the number of divisors of $n$, which is $O_epsilon(n^epsilon) = O_epsilon(max(K,L)^epsilon)$. To obtain the bound $O_epsilon(min(K,L)^epsilon)$, note that, if $L^2 < 2K/3d$, there can be at most one solutions to your equation, as two consecutive values of $k^2$ differ by at least $2 K + 1$, and $d (2 L)^2 - d L^2 = 3 d L^2$.



For $d<0$, you also have to take quadratic units in $mathbbQ(sqrtd)$, but, as there is only a logarithmic number of them in a box of given size (the group of units being isomorphic to $mathbbZ$ times bounded torsion), you still get a bound of $O_epsilon(max(K,L)^epsilon)$, and hence of $O_epsilon(min(K,L)^epsilon)$.



The only exception is given by $n=0$ and $d$ of the form $-r^2$, $r$ an integer. Then the number of solutions to $k^2+d l^2 = n$ is evidently infinite, and the number of solutions in a box is linear on the size of the box. It is easy to see that this is the case of $c=-(a+ b)/4$, $a b=-r^2$ in the equation $a (k^2+k) + b (l^2+ l) = c$. That case corresponds to $sigmain mathbbQ^2$, $m = (sigma-1)/4$ (and thus $sigmain mathbbZ^2$) in the original problem.



tl;dr a simple exercise in quadratic number fields






share|cite|improve this answer










New contributor




Nell is a new contributor to this site. Take care in asking for clarification, commenting, and answering.
Check out our Code of Conduct.






$endgroup$








  • 1




    $begingroup$
    Nell, thanks so much for your explanation !
    $endgroup$
    – Marcelo Nogueira
    4 hours ago
















6












$begingroup$

First of all, you can assume $sigmain mathbbQ$. Otherwise, the only solutions there can be are those with $l^2+l=0$ and $k^2+k=m$, and there's certainly only a bounded number of those.



Multiplying by the denominator, we see that we are being asked for a bound on the number of solutions to
$$a (k^2 + k) + b (l^2 + l) = c$$
with $k$, $l$ in dyadic intervals. We multiply both sides by $4$ and add $a+b$ to complete the squares. Thus we reduce our problem to that of bounding the number of solutions to
$$a k^2 + b l^2 = c$$
with $k$, $l$ in dyadic intervals. Multiplying by $a$ and then replacing $a k$ by $k$, we reduce our problem to that of bounding the number of solutions to
$$k^2 + d l^2 = n$$
for given $d$ and $n$,
with $k$ and $l$ in dyadic intervals $K<kleq 2 K$, $L<lleq 2 L$.
(I take the implied constant in the bound you wish is allowed to depend on $sigma$. The bound I will give will depend on $d$, though not on $n$.)



For $d>0$, the number of solutions is obviously bounded by the number of ideals of norm $n$ in the ring of integers of $mathbbQ(sqrtd)$. That number is bounded by the number of divisors of $n$, which is $O_epsilon(n^epsilon) = O_epsilon(max(K,L)^epsilon)$. To obtain the bound $O_epsilon(min(K,L)^epsilon)$, note that, if $L^2 < 2K/3d$, there can be at most one solutions to your equation, as two consecutive values of $k^2$ differ by at least $2 K + 1$, and $d (2 L)^2 - d L^2 = 3 d L^2$.



For $d<0$, you also have to take quadratic units in $mathbbQ(sqrtd)$, but, as there is only a logarithmic number of them in a box of given size (the group of units being isomorphic to $mathbbZ$ times bounded torsion), you still get a bound of $O_epsilon(max(K,L)^epsilon)$, and hence of $O_epsilon(min(K,L)^epsilon)$.



The only exception is given by $n=0$ and $d$ of the form $-r^2$, $r$ an integer. Then the number of solutions to $k^2+d l^2 = n$ is evidently infinite, and the number of solutions in a box is linear on the size of the box. It is easy to see that this is the case of $c=-(a+ b)/4$, $a b=-r^2$ in the equation $a (k^2+k) + b (l^2+ l) = c$. That case corresponds to $sigmain mathbbQ^2$, $m = (sigma-1)/4$ (and thus $sigmain mathbbZ^2$) in the original problem.



tl;dr a simple exercise in quadratic number fields






share|cite|improve this answer










New contributor




Nell is a new contributor to this site. Take care in asking for clarification, commenting, and answering.
Check out our Code of Conduct.






$endgroup$








  • 1




    $begingroup$
    Nell, thanks so much for your explanation !
    $endgroup$
    – Marcelo Nogueira
    4 hours ago














6












6








6





$begingroup$

First of all, you can assume $sigmain mathbbQ$. Otherwise, the only solutions there can be are those with $l^2+l=0$ and $k^2+k=m$, and there's certainly only a bounded number of those.



Multiplying by the denominator, we see that we are being asked for a bound on the number of solutions to
$$a (k^2 + k) + b (l^2 + l) = c$$
with $k$, $l$ in dyadic intervals. We multiply both sides by $4$ and add $a+b$ to complete the squares. Thus we reduce our problem to that of bounding the number of solutions to
$$a k^2 + b l^2 = c$$
with $k$, $l$ in dyadic intervals. Multiplying by $a$ and then replacing $a k$ by $k$, we reduce our problem to that of bounding the number of solutions to
$$k^2 + d l^2 = n$$
for given $d$ and $n$,
with $k$ and $l$ in dyadic intervals $K<kleq 2 K$, $L<lleq 2 L$.
(I take the implied constant in the bound you wish is allowed to depend on $sigma$. The bound I will give will depend on $d$, though not on $n$.)



For $d>0$, the number of solutions is obviously bounded by the number of ideals of norm $n$ in the ring of integers of $mathbbQ(sqrtd)$. That number is bounded by the number of divisors of $n$, which is $O_epsilon(n^epsilon) = O_epsilon(max(K,L)^epsilon)$. To obtain the bound $O_epsilon(min(K,L)^epsilon)$, note that, if $L^2 < 2K/3d$, there can be at most one solutions to your equation, as two consecutive values of $k^2$ differ by at least $2 K + 1$, and $d (2 L)^2 - d L^2 = 3 d L^2$.



For $d<0$, you also have to take quadratic units in $mathbbQ(sqrtd)$, but, as there is only a logarithmic number of them in a box of given size (the group of units being isomorphic to $mathbbZ$ times bounded torsion), you still get a bound of $O_epsilon(max(K,L)^epsilon)$, and hence of $O_epsilon(min(K,L)^epsilon)$.



The only exception is given by $n=0$ and $d$ of the form $-r^2$, $r$ an integer. Then the number of solutions to $k^2+d l^2 = n$ is evidently infinite, and the number of solutions in a box is linear on the size of the box. It is easy to see that this is the case of $c=-(a+ b)/4$, $a b=-r^2$ in the equation $a (k^2+k) + b (l^2+ l) = c$. That case corresponds to $sigmain mathbbQ^2$, $m = (sigma-1)/4$ (and thus $sigmain mathbbZ^2$) in the original problem.



tl;dr a simple exercise in quadratic number fields






share|cite|improve this answer










New contributor




Nell is a new contributor to this site. Take care in asking for clarification, commenting, and answering.
Check out our Code of Conduct.






$endgroup$



First of all, you can assume $sigmain mathbbQ$. Otherwise, the only solutions there can be are those with $l^2+l=0$ and $k^2+k=m$, and there's certainly only a bounded number of those.



Multiplying by the denominator, we see that we are being asked for a bound on the number of solutions to
$$a (k^2 + k) + b (l^2 + l) = c$$
with $k$, $l$ in dyadic intervals. We multiply both sides by $4$ and add $a+b$ to complete the squares. Thus we reduce our problem to that of bounding the number of solutions to
$$a k^2 + b l^2 = c$$
with $k$, $l$ in dyadic intervals. Multiplying by $a$ and then replacing $a k$ by $k$, we reduce our problem to that of bounding the number of solutions to
$$k^2 + d l^2 = n$$
for given $d$ and $n$,
with $k$ and $l$ in dyadic intervals $K<kleq 2 K$, $L<lleq 2 L$.
(I take the implied constant in the bound you wish is allowed to depend on $sigma$. The bound I will give will depend on $d$, though not on $n$.)



For $d>0$, the number of solutions is obviously bounded by the number of ideals of norm $n$ in the ring of integers of $mathbbQ(sqrtd)$. That number is bounded by the number of divisors of $n$, which is $O_epsilon(n^epsilon) = O_epsilon(max(K,L)^epsilon)$. To obtain the bound $O_epsilon(min(K,L)^epsilon)$, note that, if $L^2 < 2K/3d$, there can be at most one solutions to your equation, as two consecutive values of $k^2$ differ by at least $2 K + 1$, and $d (2 L)^2 - d L^2 = 3 d L^2$.



For $d<0$, you also have to take quadratic units in $mathbbQ(sqrtd)$, but, as there is only a logarithmic number of them in a box of given size (the group of units being isomorphic to $mathbbZ$ times bounded torsion), you still get a bound of $O_epsilon(max(K,L)^epsilon)$, and hence of $O_epsilon(min(K,L)^epsilon)$.



The only exception is given by $n=0$ and $d$ of the form $-r^2$, $r$ an integer. Then the number of solutions to $k^2+d l^2 = n$ is evidently infinite, and the number of solutions in a box is linear on the size of the box. It is easy to see that this is the case of $c=-(a+ b)/4$, $a b=-r^2$ in the equation $a (k^2+k) + b (l^2+ l) = c$. That case corresponds to $sigmain mathbbQ^2$, $m = (sigma-1)/4$ (and thus $sigmain mathbbZ^2$) in the original problem.



tl;dr a simple exercise in quadratic number fields







share|cite|improve this answer










New contributor




Nell is a new contributor to this site. Take care in asking for clarification, commenting, and answering.
Check out our Code of Conduct.









share|cite|improve this answer



share|cite|improve this answer








edited 3 hours ago





















New contributor




Nell is a new contributor to this site. Take care in asking for clarification, commenting, and answering.
Check out our Code of Conduct.









answered 6 hours ago









NellNell

614




614




New contributor




Nell is a new contributor to this site. Take care in asking for clarification, commenting, and answering.
Check out our Code of Conduct.





New contributor





Nell is a new contributor to this site. Take care in asking for clarification, commenting, and answering.
Check out our Code of Conduct.






Nell is a new contributor to this site. Take care in asking for clarification, commenting, and answering.
Check out our Code of Conduct.







  • 1




    $begingroup$
    Nell, thanks so much for your explanation !
    $endgroup$
    – Marcelo Nogueira
    4 hours ago













  • 1




    $begingroup$
    Nell, thanks so much for your explanation !
    $endgroup$
    – Marcelo Nogueira
    4 hours ago








1




1




$begingroup$
Nell, thanks so much for your explanation !
$endgroup$
– Marcelo Nogueira
4 hours ago





$begingroup$
Nell, thanks so much for your explanation !
$endgroup$
– Marcelo Nogueira
4 hours ago











Marcelo Nogueira is a new contributor. Be nice, and check out our Code of Conduct.









draft saved

draft discarded


















Marcelo Nogueira is a new contributor. Be nice, and check out our Code of Conduct.












Marcelo Nogueira is a new contributor. Be nice, and check out our Code of Conduct.











Marcelo Nogueira is a new contributor. Be nice, and check out our Code of Conduct.














Thanks for contributing an answer to MathOverflow!


  • Please be sure to answer the question. Provide details and share your research!

But avoid


  • Asking for help, clarification, or responding to other answers.

  • Making statements based on opinion; back them up with references or personal experience.

Use MathJax to format equations. MathJax reference.


To learn more, see our tips on writing great answers.




draft saved


draft discarded














StackExchange.ready(
function ()
StackExchange.openid.initPostLogin('.new-post-login', 'https%3a%2f%2fmathoverflow.net%2fquestions%2f325448%2fupper-bound-for-the-number-of-solutions-of-a-diophantine-equation%23new-answer', 'question_page');

);

Post as a guest















Required, but never shown





















































Required, but never shown














Required, but never shown












Required, but never shown







Required, but never shown

































Required, but never shown














Required, but never shown












Required, but never shown







Required, but never shown







Popular posts from this blog

Identifying “long and narrow” polygons in with PostGISlength and width of polygonWhy postgis st_overlaps reports Qgis' “avoid intersections” generated polygon as overlapping with others?Adjusting polygons to boundary and filling holesDrawing polygons with fixed area?How to remove spikes in Polygons with PostGISDeleting sliver polygons after difference operation in QGIS?Snapping boundaries in PostGISSplit polygon into parts adding attributes based on underlying polygon in QGISSplitting overlap between polygons and assign to nearest polygon using PostGIS?Expanding polygons and clipping at midpoint?Removing Intersection of Buffers in Same Layers

Masuk log Menu navigasi

อาณาจักร (ชีววิทยา) ดูเพิ่ม อ้างอิง รายการเลือกการนำทาง10.1086/39456810.5962/bhl.title.447410.1126/science.163.3863.150576276010.1007/BF01796092408502"Phylogenetic structure of the prokaryotic domain: the primary kingdoms"10.1073/pnas.74.11.5088432104270744"Towards a natural system of organisms: proposal for the domains Archaea, Bacteria, and Eucarya"1990PNAS...87.4576W10.1073/pnas.87.12.4576541592112744PubMedJump the queueexpand by handPubMedJump the queueexpand by handPubMedJump the queueexpand by hand"A revised six-kingdom system of life"10.1111/j.1469-185X.1998.tb00030.x9809012"Only six kingdoms of life"10.1098/rspb.2004.2705169172415306349"Kingdoms Protozoa and Chromista and the eozoan root of the eukaryotic tree"10.1098/rsbl.2009.0948288006020031978เพิ่มข้อมูล